Administración     

Olimpiadas de Matemáticas
Página de preparación y problemas

OME Local
OME Andaluza
OME Nacional
OIM
IMO
EGMO
USAMO
ASU
APMO
OMCC
Retos UJA
Selector
La base de datos contiene 2748 problemas y 1042 soluciones.

XL Olimpiada Iberoamericana de Matemáticas — 2025

Sesión 1 —  25 de septiembre de 2025

Problema 2716
Una sucesión de números reales $a_1,a_2,\ldots$ se llama mapuche si $a_1\gt 0$ y además para todo $n\geq 2$ se tiene que \[a_1a_2\cdots a_n=a_1+a_2+\ldots+a_{n-1}.\] ¿Cuál es la máxima cantidad de enteros que puede tener una sucesión mapuche?

Nota. El producto tiene $n$ factores y la suma tiene $n-1$ sumandos.

pistasolución 1info
Pista. Demuestra que $1\lt a_n\lt 2$ para todo $n\geq 5$.
Solución. Observemos que $a_2=1$ y además podemos expresar \[a_1a_2\cdots a_{n+1}=a_1+a_2+\ldots +a_n=a_1a_2\cdots a_n+a_n,\] de donde obtenemos que \[a_{n+1}=1+\frac{1}{a_1a_2\cdots a_{n-1}}\quad\text{para todo }n\geq 2.\qquad(\star)\] Cómo todos los $a_n$ son positivos, esto nos dice que que $a_n\gt 1$ para todo $n\geq 3$. Además, tenemos que $a_1a_2a_3=a_1+1\gt 1$, luego deducimos que $a_1a_2\cdots a_{n-1}\gt 1$ para todo $n\geq 4$. Por lo tanto, $(\star)$ nos asegura que $1\lt a_n\lt 2$ para todo $n\geq 5$. Así, a lo sumo hay cuatro términos que son enteros en toda sucesión mapuche.

Para resolver así el problema, será suficiente encontrar una sucesión mapuche con cuatro términos enteros, para lo que tomamos $a_1=1, condición inicial que nos lleva a que $a_2=1$, $a_3=2$ y $a_4=2$.

Si crees que el enunciado contiene un error o imprecisión o bien crees que la información sobre la procedencia del problema es incorrecta, puedes notificarlo usando los siguientes botones:
Informar de error en enunciado Informar de procedencia del problema
Problema 2717
Se tiene un tablero $n\times n$ dividido en $n^2$ casillas con $n\geq 3$. Inicialmente, se elige una casilla y se colocan en ella $n^2$ monedas. Un movimiento consiste en elegir una casilla que contenga al menos dos monedas y desplazar dos de ellas hacia dos casillas que sean simétricas con respecto a la casilla elegida y compartan al menos un vértice con ella. En la figura se muestran los cuatro tipos de movimientos posibles.

Si después de varios movimientos rsulta que en cada casilla del tablero hay exactamente una moneda, demostrar que la cantidad de movimientos realizados del tipo $3$ es igual a la cantidad de movimientos realizados del tipo $4$.

imagen
pistasolución 1info
Pista. Hay invariantes relacionados con las sumas de las coordenadas de las monedas y con las sumas de los cuadrados de dichas coordenadas.
Solución. Pongamos coordenadas $(x_i,y_i)$ a cada moneda para $1\leq i\leq n^2$, siendo $(0,0)$ la casilla inferior izquierda y $(n^2,n^2)$ la de la casilla superior derecha. Tenemos entonces que las sumas \[S_x=\sum_{i=1}^{n^2}x_i,\qquad S_y=\sum_{i=1}^{n^2}y_i\] no cambian al hacer movimientos de cualquiera de los cuatro tipos. Si inicialmente las $n^2$ monedas se encuentran (todas ellas) en la casilla $(a,b)$, entonces comparando los valores de $S_x$ y $S_y$ con el momento en que todas las casillas tienen una moneda, obtenemos las igualdades \[an^2=bn^2=n(1+2+\ldots+n)=\tfrac{1}{2}n^2(n+1),\] de donde $a=b=\frac{n+1}{2}$. Esto nos dice que $n$ es impar y que la única posible posición inicial es la casilla central. Por lo tanto, supondremos que $n=2m+1$ para cierto entero positivo $m$ y cambiamos las coordenadas para que $(0,0)$ sea la casilla central, de forma que las casillas del tablero tienen coordenadas enteras $(x,y)$ con $-m\leq x,y\leq m$.

Consideramos ahora las siguientes sumas \[A=\sum_{i=1}^{n^2}x_i^2,\quad B=\sum_{i=1}^{n^2}y_i^2,\quad C=\sum_{i=1}^{n^2}(x_i+y_i)^2,\quad D=\sum_{i=1}^{n^2}(x_i-y_i)^2.\] Los movimientos de tipo II, III y IV incrementan $A$ en una unidad (puesto que $(x-1)^2+(x+1)^2=x^2+x^2+2$) mientras que los de tipo I lo dejan invariante. Análogamente, los movimientos de tipo I, III y IV incrementan $B$ en una unidad mientras que los de tipo II lo dejan invariante. La situación inicial es $A=B=0$ y la final también cumple $A=B$, luego tenemos que haber hecho tantos movimientos de tipos II-III-IV como movimientos de tipos I-III-IV. De aquí deducimos que se han hecho tantos movimientos de tipo I como de tipo II en el proceso (no nos piden esto pero será útil).

Los movimientos de tipo I y II incrementan $C$ y $D$ en $2$ unidades; los de tipo III incrementan $C$ en $4$ unidades y dejan $D$ invariante, y finalmente los de tipo IV dejan $C$ invariante e incrementan $D$ en $4$ unidades. Inicialmente tenemos $C=D=0$ y en el estado final tenemos también $C=D$ por simetría. Como se ha realizado el mismo número de movimientos de tipo I que de tipo II, también tendrá que haberse realizado el mismo número de movimientos de tipo III que de tipo IV.

Nota. El hecho de considerar las sumas y las sumas de los cuadrados está inspirado por la media y la varianza de los datos. Observemos que los cuatro tipos de movimientos dejan invariante las medias de las coordenadas (es decir, no cambian el centro de masa de las monedas), pero las van dispersando aumentando siempre la desviación. En la solución, hemos ido midiendo las varianzas de las desviaciones en las direcciones de los ejes y en las direcciones de las bisectrices de los cuadrantes.

Si crees que el enunciado contiene un error o imprecisión o bien crees que la información sobre la procedencia del problema es incorrecta, puedes notificarlo usando los siguientes botones:
Informar de error en enunciado Informar de procedencia del problema
Problema 2718
Sean $b$ y $n$ enteros positivos con $b\geq 2$. Se define $s_b(n)$ como la suma de las cifras de $n$ expresado en base $b$. ¿Existe algún entero $n\geq 2$ tal que \[s_2(n)\geq s_3(n)\geq\ldots\geq s_{2025}(n)?\]

Nota. Las cifras de $n$ expresado en base $b$ son los números enteros $a_0,a_1,\ldots,a_k$ tales que $n=a_0+a_1b+a_2b^2+\ldots+a_kb^k$ con $a_k\neq 0$ y $0\leq a_i\leq b-1$ para todo $i\in\{0,1,\ldots,k\}$.

Sin pistas
Sin soluciones
info
Si crees que el enunciado contiene un error o imprecisión o bien crees que la información sobre la procedencia del problema es incorrecta, puedes notificarlo usando los siguientes botones:
Informar de error en enunciado Informar de procedencia del problema

Sesión 2 —  26 de septiembre de 2025

Problema 2719
Encontrar todas las parejas de números primos $(p,q)$ con $p\gt q\gt 1$, tales que \[(p-q-1)^3+(p-q)^3+\ldots+(p-1)^3+p^3+\ldots+(p+q)^3+(p+q+1)^3=(3pq)^2.\]

Nota. El miembro de la izquierda de la igualdad tiene $2q+3$ sumandos, los cuales son cubos de números consecutivos.

pistasolución 1info
Pista. Utiliza la igualdad \[1^3+2^3+\ldots+n^3=(1+2+\ldots+n)^2=\frac{n^2(n+1)^2}{4}\] para quitar los puntos suspensivos.
Solución. Utilizaremos la fórmula conocida para la suma de los primeros $n$ cubos \[1^3+2^3+\ldots+n^3=(1+2+\ldots+n)^2=\frac{n^2(n+1)^2}{4}.\] La suma de la izquierda del enunciado se puede expresar como la anterior suma para $n=p+q+1$ menos esa suma para $n=p-q-2$. Por tanto, podemos expresar la ecuación del enunciado como \[\frac{(p+q+1)^2(p+q+2)^2}{4}-\frac{(p-q-2)^2(p-q-1)^2}{4}=9p^2q^2.\] Podemos factorizar el miembro de la derecha fácilmente al tratarse de una diferencia de cuadrados y, tras simplificar, llegamos a la ecuación equivalente \[(2q+3)(p^2+3q+q^2+2)=9pq^2.\qquad(\star)\] Si ninguno de los primos es $3$, entonces $p^2\equiv 1\pmod 3$ y $q^2\equiv 1\pmod 3$, luego $p^2+3q+q^2+2\equiv 1\pmod 3$ y $2q+3\not\equiv 0\pmod 3$, luego la ecuación $(\star)$ no tiene soluciones. Tenemos entonces dos casos posibles:
  • Si $q=3$, la ecuación $(\star)$ queda $p^2-9p+20=0$ tras simplificar, que tiene por soluciones $p=4$ y $p=5$. Nos quedamos únicamente con $p=5$ ya que debe tratarse de un número primo.
  • Si $p=3$, entonces $(\star)$ queda $2q^3-18q^2+31q+33=0$. Las únicas posibles soluciones que son números primos son $q=3$ y $q=33$ (divisores primos del término independiente), pero ninguna de ellas cumple la ecuación.

Deducimos que la única solución es $(p,q)=(5,3)$.

Si crees que el enunciado contiene un error o imprecisión o bien crees que la información sobre la procedencia del problema es incorrecta, puedes notificarlo usando los siguientes botones:
Informar de error en enunciado Informar de procedencia del problema
Problema 2720
El triángulo $ABC$ es acutángulo con $AB\lt AC$. Sean $\omega$ la circunferencia inscrita del triángulo $ABC$ y $\Gamma$ su circunferencia circunscrita. Sea $D$ el punto de tangencia de $\omega$ con el lado $BC$ y sea $L$ el punto de $\omega$ diametralmente opuesto a $D$. La recta $AL$ corta al lado $BC$ en el punto $E$. Sea $N$ el punto medio del arco $BC$ de $\Gamma$ que contiene a $A$. La recta $NL$ corta de nuevo a $\omega$ en el punto $K$. Demostrar que los puntos $A,N,E,K$ están en una misma circunferencia.
Sin pistas
Sin soluciones
info
Si crees que el enunciado contiene un error o imprecisión o bien crees que la información sobre la procedencia del problema es incorrecta, puedes notificarlo usando los siguientes botones:
Informar de error en enunciado Informar de procedencia del problema
Problema 2721
Un sultán tiene capturados a $23$ magos y les propone un juego para dejarlos libres. El sultán les dice que va a construir $11$ pozos, numerados del $1$ al $11$, y una torre. Dentro de cada pozo pondrá a dos magos y pondrá al restante en la torre. A cada mago en los pozos le pondrá un sombrero de uno de cuatro colores (conocidos por todos), y al de la torre le pondrá un sombrero de uno de $2025$ colores (distintos de los otros cuatro y conocidos por todos). Ningún mago sabrá el color de su sombrero. Una vez dentro del pozo, cada mago sabrá el número del pozo en el que está; además, verá únicamente el sombrero del mago de la torre y el del mago que el que compartirá pozo. El mago de la torre conocerá el número de cada pozo y podrá ver los sombreros de todos los demás magos.

En un determinado momento, el sultán dará la orden y, simultáneamente, cada mago dirá El color de mi sombrero es X, donde X es el color que quiera. Si al menos un mago dice una frase verdadera, todos los magos ganan y son libres; en otro caso, pierden. Antes de ser puestos en sus lugares y de recibir sus sombreros, los magos dispondrán de un tiempo para planear una estrategia, pero no podrán comunicarse después de esto. ¿Pueden asegurar la victoria sin importar lo que haga el sultán?

pistasolución 1info
Pista. Piensa que para determinar un número entre $1$ y $2025$ hacen falta $11$ dígitos $0$ o $1$ en binario, que coincide con el número de pozos. Elabora una estrategia en la que, si el mago de la torre no acierta, entonces necesariamente uno de los magos de algún pozo tiene que acertar.

Piensa que cada mago dice su respuesta únicamente en función de los colores de los sombreros que ve; el resto de magos no le proporciona información con sus respuestas ya que todos las dan simultáneamente.

Solución. La respuesta es afirmativa y para demostrarlo describiremos la estrategia que pueden seguir los magos. La idea principal es codificar cada uno de los 2025 colores del mago de la torre con un número del $0$ al $2024$ en base $2$, lo que supone $11$ dígitos $0$ o $1$ ya que $2^{11}=2048\gt 2025$. Por otro lado, a cada uno de los cuatro colores de los magos de los pozos le asignamos un número del $0$ al $3$ módulo $4$. Con esto en mente, cada mago responde lo siguiente.
  • El mago de la torre forma un número binario de $11$ dígitos, siendo el $i$-ésimo dígito igual a $0$ si los colores de los dos magos del pozo $i$ tienen la misma paridad y $1$ si estos tienen distinta paridad. Identifica este número con uno de los $2025$ colores y responde ese color. Si el número formado no se corresponde con ningún color (está entre $2025$ y $2047$), entonces responde un color aleatorio.
  • En el pozo $i$, cada mago mira el dígito $i$-ésimo del número binario asignado al color del mago de la torre. Si el dígito es $0$, entonces el primer mago responde el color del segundo mago más $1$ y el segundo mago el color del primer mago más $1$ (módulo $4$); si, por el contrario, el dígito del mago de la torre es $1$, entonces el primer mago responde el mismo color del segundo mago y el segundo mago el color del primer mago más $2$ (módulo $4$). Esta estrategia presupone que los magos del pozo han elegido un orden entre ellos previamente.

De esta forma, si el mago de la torre no acierta, es porque necesariamente hay un pozo en el que la paridad de los colores de sus dos magos no coincide con la indicada por el correspondiente dígito del mago de la torre. La estrategia de los magos de dicho pozo barre todos los casos posibles en que esto ocurre, luego alguno de los dos debe acertar su color. Observemos que hay ocho posibles combinaciones de colores con igual/distinta paridad en el pozo, pero cada mago del pozo ve el color del otro mago, lo que realmente reduce las combinaciones a dos y cada mago explora una de ellas.

Si crees que el enunciado contiene un error o imprecisión o bien crees que la información sobre la procedencia del problema es incorrecta, puedes notificarlo usando los siguientes botones:
Informar de error en enunciado Informar de procedencia del problema
José Miguel Manzano © 2010-2025. Esta página ha sido creada mediante software libre